Xem bài viết đơn
Old 25-12-2010, 04:07 PM   #2
Lan Phuog
+Thành Viên Danh Dự+
 
Lan Phuog's Avatar
 
Tham gia ngày: Mar 2010
Đến từ: Thái Bình
Bài gởi: 564
Thanks: 289
Thanked 326 Times in 182 Posts
Trích:
Nguyên văn bởi jakelong View Post
Các pác Cm giùm em bài này với cách sử dụng Bất Đẳng Thức AM-GM Cho 2 Số Nhé
Cho $x,y,z > 0 $ và $xyz=1 $ Cm:
$(1+\frac{x}{y})(1+\frac{y}{z})(1+\frac{z}{x})\geq (1+x)(1+y)(1+z) $
Nhân khai triển và rút gọn thì bdt tương đương với:
$\frac{x}{z}+\frac{z}{y}+\frac{y}{x}\ge xy+yz+zx $
Sử dụng AM-GM như sau:
$\frac{x}{z}+\frac{z}{y}+\frac{z}{y}\ge 3\sqrt[3]{\frac{zx}{y^2}}=3zx $
Làm mấy cái tương tự rồi cộng vào thu được đpcm!
[RIGHT][I][B]Nguồn: MathScope.ORG[/B][/I][/RIGHT]
 
Lan Phuog is offline   Trả Lời Với Trích Dẫn
The Following 2 Users Say Thank You to Lan Phuog For This Useful Post:
avip (25-12-2010), jakelong (25-12-2010)
 
[page compression: 8.95 k/10.05 k (10.97%)]